Barron's GRE, 18th Edition (2009)

Part 2. VERBAL ABILITY: TACTICS, REVIEW, AND PRACTICE

Chapter 7. Reading Comprehension Questions

GRE reading comprehension questions test your ability to understand what you read — both content and technique. Each verbal section on the GRE CAT includes two to four relatively short passages, each passage followed by two to four questions. A passage may deal with the sciences (including medicine, botany, zoology, chemistry, physics, geology, astronomy); the humanities (including art, literature, music, philosophy, folklore); or the social sciences (including history, economics, sociology, government). Some passages are strictly objective, explaining or describing a phenomenon or process neutrally. Others reflect a particular bias or point of view: the author is trying to convince the reader to share his or her opinion about the subject being discussed.

The GRE tends to take its reading passages from The New York Review of Books, from prestigious university presses (Harvard, Princeton, Oxford), from scholarly journals. Often the test makers hit academically “hot” topics — biodiesel fuels, plate tectonics, damage to the ozone layer, Arthurian romance, the status of women’s literature — that have aroused controversy over the past several decades. Frequently they edit these passages to make them more demanding both in vocabulary level and in grammatical complexity.

Some of the reading comprehension questions on the GRE are factual, asking you about specific details in the passages. Others ask you to interpret the passages, to make judgments about them. Still others ask you to recognize various techniques used by the authors or possible applications of their ideas to other circumstances. Some questions include lengthy and complex statements, as lengthy and complex as any sentences in the passage. Read the questions closely, as closely as you read the text. Be sure, in answering reading comprehension questions, that you read all the answer choices before deciding which is correct.

The reading comprehension questions following each passage are not arranged in order of difficulty. They are arranged to reflect the way the passage’s content is organized. A question based on information found at the beginning of the passage generally will come before a question based on information at the passage’s end.

Testing Tactics

 First Read the Question, Then Read the Passage

In responding to reading comprehension passages on the CAT, you generally will have to consider more material than can fit conveniently on a single screen. You will confront a split screen similar to the one on this page. On one-half of the screen you will see the question you must answer; on the other you will see a segment of the passage under consideration. You will have to scroll through the passage in order to read the text in its entirety. (For a more comprehensive explanation of scrolling through passages, see Chapter 2.)

Under these conditions, clearly only one tactic works: first read the question, then read the passage.

1.            Read the question carefully, so that you are sure you understand what it is asking. Decide whether it is asking about a specific, readily identifiable detail within the passage, or whether it is asking about the passage as a whole. Note any key words in the question that may help you spot where the answer may be found.

2.            Next, turn to the passage. Read as rapidly as you can with understanding, but do not force yourself. Do not worry about the time element. If you worry about not finishing the test, you will begin to take shortcuts and miss the correct answer in your haste.

3.            As you read the opening sentences, try to anticipate what the passage will be about. Whom or what is the author talking about? What, in other words, is the topic of this passage?

4.            As you scroll through the passage, think about what kind of writing this is. What is the author trying to do?

Is the author trying to explain some aspect of the topic?

Is the author trying to describe some aspect of the topic?

Is the author trying to argue or debate some aspect of the topic?

What does the author feel about this topic? What audience is the author addressing here? Answering these questions will give you a sense of the passage as a whole.

5.            Use your scratch paper intelligently. Take brief notes of important words or phrases in different paragraphs so that you can scroll back to them quickly when you want to verify an answer choice. You may also want to note key words in question stems (words like EXCEPT and LEAST, which the test-makers capitalize for emphasis, and that restrict your answer choice).

6.            Your first scrolling through the passage should give you a general impression of the scope of the passage and of the location of its major subdivisions. In order to answer the question properly, you must go back to the passage to verify your answer choice. Do not rely on your memory. Above all, do not rely on anything you may have learned from your reading or courses about the topic of this passage. Base your answer on what this passage says, not on what you know from other sources.

 Learn to Spot the Major Reading Question Types

Just as it will help you to know the common types of analogies found on the GRE, it will also help you to familiarize yourself with the major types of reading questions on the test.

If you can recognize just what a given question is asking for, you will be better able to tell which reading tactic to apply.

Here are seven categories of reading questions you are likely to face:

1.            Main Idea Questions that test your ability to find the central thought of a passage or to judge its significance often take one of the following forms:

The main point of the passage is to…
The passage is primarily concerned with…
The author’s primary purpose in this passage is to…
The chief theme of the passage can best be described as…
Which of the following titles best states the central idea of the passage?
Which of the following statements best expresses the main idea of the passage?

2.            Finding Specific Details Questions that test your ability to understand what the author states explicitly are often worded:

According to the author,…
The author states all of the following EXCEPT…
According to the passage, which of the following is true of the…
The passage supplies information that would answer which of the following questions?
Which of the following statements is (are) best supported by the passage?
Which of the following is NOT cited in the passage as evidence of…?

3.            Drawing Inferences Questions that test your ability to go beyond the author’s explicit statements and see what these statements imply may be worded:

It can be inferred from the passage that…
The author implies that…
The passage suggests that…
Which of the following statements about…can be inferred from the passage?

4.            Application to Other Situations Questions that test your ability to recognize how the author’s ideas might apply to other situations often are worded:

With which of the following statements would the author of the passage be most likely to agree?
With which of the following aphorisms would the author be in strongest agreement?
The author’s argument would be most weakened by the discovery of which of the following?
The author’s contention would be most clearly strengthened if which of the following were found to be true?
Which of the following examples could best be substituted for the author’s example of…?
Which of the following statements would be most likely to begin the paragraph immediately following the passage?
The author is most probably addressing which of the following audiences?

5.            Tone/Attitude Questions that test your ability to sense an author’s emotional state often take the form:

The author’s attitude toward the problem can best be described as…
The author regards that idea that…with…
The author’s tone in the passage is that of a person attempting to…
Which of the following best describes the author’s tone in the passage?

6.            Technique Questions that test your ability to recognize a passage’s method of organization or technique often are worded:

Which of the following best describes the development of this passage?
In presenting the argument, the author does all of the following EXCEPT…
The relationship between the second paragraph and the first paragraph can best be described as…
In the passage, the author makes the central point primarily by…
The organization of the passage can best be described as…

7.            Determining the Meaning of Words from Their Context Questions that test your ability to work out the meaning of unfamiliar words from their context often are worded:

As it is used in the passage, the term…can best be described as…
The phrase…is used in the passage to mean that…
As used by the author, the term…refers to…
The author uses the phrase…to describe…

 When Asked to Find the Main Idea, Be Sure to Check the Opening and Summary Sentences of Each Paragraph

The opening and closing sentences of a paragraph are key sentences for you to read. They can serve as guideposts, pointing out the author’s main idea.

When you are asked to determine a passage’s main idea, always check the opening and summary sentences of each paragraph. Authors typically provide readers with a sentence that expresses a paragraph’s main idea succinctly. Although such topic sentences may appear anywhere in the paragraph, readers customarily look for them in the opening or closing sentences.

Note that in GRE reading passages topic sentences are sometimes implied rather than stated directly. If you cannot find a topic sentence, ask yourself these questions:

1.            Who or what is this passage about?

(The subject of the passage can be a personplace, or thing. It can be something abstract, such as an idea. It can even be a process, or something in motion, for which no single-word synonym exists.)

2.            What aspect of this subject is the author talking about?

3.            What is the author trying to get across about this aspect of the subject?

(Decide the most important thing that is being said about the subject.
Either the subject must be doing something, or something is being done to it.)

Read the following natural science passage and apply this tactic.

         According to Wilson1, only when we are able to apply the same parameters 
         and mathematical principles to weighing both troops of rhesus 
         macaques and termite colonies will a unified science of sociobiology 
Line finally exist. While recognizing that many of his colleagues question such 
  (5) an outcome, Wilson, one of sociobiology’s leading proponents, finds himself 
        simultaneously more and more struck by the functional similarities 
         that characterize both insect and vertebrate societies and less concerned 
         with the structural differences that divide them to such an apparently 
         irreconcilable degree. Thus, he freely compares termites and macaques, 
 (10) pointing out numerous likenesses between them. Both societies are territorial: 
        they occupy a particular home range, which they defend against 
         intruders. Likewise, both are cooperative: members organize themselves 
         into working groups that observe a clearly-defined division of labor. In 
         addition, members of both groups can convey to each other a range of 
 (15) basic emotions and personal information: animosity, fright, hunger, rank 
         within a particular caste, and ability to reproduce. Wilson readily 
         concedes that, from a specialist’s perspective, such a likeness may at first 
         appear superficial, even unscientifically glib. Nonetheless, in this eminent 
         scholar’s judgment, “it is out of such deliberate oversimplification that the 
 (20) beginnings of a general theory are made.”

Now look at a typical main idea question on this passage.

EXAMPLE

Which of the following best summarizes the author’s main point?

(A) Facile and simplistic comparisons of animal societies could damage the prospects for the establishment of a unified science of sociobiology.

(B) It is necessary to study both biology and sociology in order to appreciate how animals as different as termites and rhesus macaques can be said to resemble each other.

(C) The majority of animal species arrange themselves in societies whose patterns of group behavior resemble those of human societies.

(D) It is worthwhile noting that animals as dissimilar as termites and rhesus monkeys observe certain analogous and predictable behavior patterns.

(E) An analysis of the ways in which insect and vertebrate societies resemble one another could supply the foundation for a unified science of sociobiology.

Look at the opening and summary sentences of the passage: “only when we are able to apply the same parameters and mathematical principles to weighing both troops of rhesus macaques and termite colonies will a unified science of sociobiology finally exist…it is out of such deliberate oversimplification that the beginnings of a general theory are made.” First, is there a person, place, thing, idea, or process that is common to both sentences? Are there any words in the last sentence that repeat something in the first? A general theory repeats the idea of a unified science of sociobiology. The paragraph’s subject seems to be the unified science of sociobiology. Note as well the words pointing to expectations for the future — will…finally existbeginnings. The tone of both sentences appears positive: when certain conditions are met, then, in Wilson’s view, a specific result will follow — we will have a unified science or general theory of sociobiology. This result, however, is not guaranteed; it can come about only if the conditions are met.

Now turn to the answer choices. What does Choice A say about a unified science of sociobiology? It states some things could make it less likely, not more likely, to come about. Choice A is incorrect; it contradicts the passage’s sense that a unified science of sociobiology is a likely outcome. Choices B, C, and D also may be incorrect: not one of them mentions a unified science of sociobiology. On closer inspection, Choice B proves incorrect: it makes an unsupported statement that one needs biological and sociological education to understand the resemblances between insects and vertebrates. Choice C also proves incorrect: it goes far beyond what the passage actually states. Where the passage speaks in terms of termites and rhesus macaques, Choice C speaks in terms of the majority of animal species and extends the comparison to include humans as well. Choice D, while factually correct according to the passage, is incorrect because it is too narrow in scope. It ignores the author’s main point; it fails to include Wilson’s interest in the possibility that a study of such similar patterns of behavior might lead to a general theory of sociobiology. The correct answer is Choice E. It is the only statement that speaks of a unified science of sociobiology as a likely possibility.

 When Asked to Choose a Title, Watch Out for Choices That Are Too Specific or Too Broad

A paragraph has been defined as a group of sentences revolving around a central theme. An appropriate title for a paragraph, therefore, must express this central theme that each of the sentences in the paragraph develops. It should be neither too broad nor too narrow in scope; it should be specific and yet comprehensive enough to include all the essential ideas presented by the sentences. A good title for a passage of two or more paragraphs should express the thoughts of ALL the paragraphs.

When you are trying to select the best title for a passage, watch out for words that come straight out of the passage. They may not always be your best choice.

This second question on the sociobiology passage is a title question. Note how it resembles questions on the passage’s purpose or main idea.

EXAMPLE

Which of the following is the best title for the passage?

(A) Deceptive Comparisons: Oversimplification in Biological Research

(B) An Uncanny Likeness: Termites and Rhesus Macaques

(C) Structural Dissimilarities Between Insects and Vertebrates

(D) Arguments Against a Science of Sociobiology

(E) Sociobiology: Intimations of a General Theory

Choice A is incorrect: it is at once too narrow and too broad. It is too narrow in that the passage refers to oversimplification only in passing; it does not have oversimplification as its subject. It is too broad in that the passage emphasizes sociobiology, not the whole realm of biological research. It is also misleading; the passage never asserts that the deliberate oversimplification of the comparison between termites and macaques is intended to deceive.

Choice B is incorrect: it is too narrow. True, the author discusses the resemblance between termite and macaque societies; however, this likeness is not his subject. He discusses it to provide an example of the sort of comparison that may lay the groundwork for a potential science of sociobiology.

Choice C is also incorrect because it is not inclusive enough. It fails to mention the potential science of sociobiology. In addition, while the passage refers to structural differences between insect and vertebrate societies, it stresses structural similarities, not structural dissimilarities.

Choices D and E both mention the theory of sociobiology. Which is the better title for the piece? Clearly, Choice E: the author is not arguing against the potential science of sociobiology; he is reporting Wilson’s opinions concerning the likelihood of sociobiology’s emergence as a unified science. Thus, he finds in the termite-macaque comparison intimations or hints of an incipient general theory.

 When Asked to Determine Questions of Attitude, Mood, or Tone, Look for Words That Convey Emotion, Express Values, or Paint Pictures

In determining the attitude, mood, or tone of an author, examine the specific diction used. Is the author using adjectives to describe the subject? If so, are they words like fragranttranquilmagnanimous — words with positive connotations? Or are they words like fetidruffledstingy — words with negative connotations?

When we speak, our tone of voice conveys our mood — frustrated, cheerful, critical, gloomy, angry. When we write, our images and descriptive phrases get our feelings across.

The next model question on the Wilson passage is an attitude question. Note the range of feelings in the answer choices.

EXAMPLE

According to the author, Wilson’s attitude toward the prospect of a unified theory in sociobiology can best be characterized as which of the following?

(A) Unconditional enthusiasm

(B) Cautious optimism

(C) Unbiased objectivity

(D) Resigned acquiescence

(E) Strong displeasure

How does Wilson feel about the possibility of a unified theory of sociobiology? The answer choices range from actively negative (strong displeasure) to actively positive (unconditional enthusiasm), with passively negative (resigned acquiescence), neutral (unbiased objectivity), and guardedly positive (cautious optimism) in between.

Wilson’s attitude toward the possibility of a unified theory of sociobiology is implicit in the author’s choice of words. It is clear that Wilson views this possibility positively; the whole thrust of his argument is that the current studies of the similarities between insect and vertebrate societies could mark the beginnings of such a unified theory and that the specialist should not dismiss these studies as glib or simpleminded. Note in the second sentence how the author describes Wilson as a leading proponent or champion of sociobiology, someone whose feelings about the field are by definition positive.

Wilson is certainly not unhappy or strongly displeased with this potential unified theory, nor is he merely long-suffering or resigned to it. Similarly, he is not unbiased and objective about it; he actively involves himself in arguing the case for sociobiology. Thus, you can eliminate Choices C, D, and E. But how do you decide between the two positive terms, enthusiasm and optimism, Choice A and Choice B? To decide between them, you must look carefully at the adjectives modifying them. Is Wilson’s enthusiasm unqualified or unconditional? You may think so, but look again. The opening sentence states a basic condition that must be met before there can be a unified science of sociobiology: the same parameters and mathematical principles must be used to analyze insect and vertebrate societies. Though a proponent of sociobiology, Wilson is first and foremost a scientist, one who tests hypotheses and comes to logical conclusions about them. Unconditional enthusiasm seems to overstate his attitude.

Choice A appears incorrect. What of Choice B? Is Wilson’s optimism cautious or guarded? Yes. According to the passage, Wilson is aware that specialists may well find fault with the sociobiologist’s conclusions; the passage uses terms that convey values, first the negative “superficial, even unscientifically glib” to suggest the specialist’s negative attitude towards sociobiology, then the positive “deliberate” to convey Wilson’s own more positive response. The correct answer is Choice B.

 When Asked About Specific Details in the Passage, Spot Key Words in the Question and Scan the Passage to Find Them (or Their Synonyms)

In developing the main idea of a passage, a writer will make statements to support his or her point. To answer questions about such supporting details, you must find a word or group of words in the passage supporting your choice of answer. The words “according to the passage” or “according to the author” should focus your attention on what the passage explicitly states. Do not be misled into choosing an answer (even one that makes good sense) if you cannot find it supported by the text.

Detail questions often ask about a particular phrase or line. In such cases, use the following technique:

1.            Look for key words (nouns or verbs) in the answer choices.

2.            Scroll through the passage, looking for those key words or their synonyms. (This is scanning. It is what you do when you look up someone’s number in the phone directory.)

3.            When you find a key word or its synonym in a sentence, reread that sentence to make sure the test makers haven’t used the original wording to mislead you.

Read the following brief passage and apply this tactic.

            What is involved in the process of visual recognition? First, like computer 
        data, visual memories of an object must be stored; then, a mechanism 
        must exist for them to be retrieved. But how does this process work? 
Line The eye triggers the nerves into action. This neural activity constructs a 
  (5) picture in the brain’s memory system, an internal image of the object 
         observed. When the eye once again confronts that object, the object is 
         compared with its internal image; if the two images match, recognition 
         takes place.

        Among psychologists, the question as to whether visual recognition is 
  (10) a parallel, single-step operation or a sequential, step-by-step one is the 
         subject of much debate. Gestalt psychologists contend that objects are 
         perceived as wholes in a parallel operation: the internal image is matched 
         with the retinal impression in one single step. Psychologists of other 
         schools, however, suggest the opposite, maintaining that the individual 
(15) features of an object are matched serially with the features of its internal 
         image. Some experiments have demonstrated that the more well-known 
         an object is, the more holistic its internal image becomes, and the more 
         parallel the process of recognition tends to be. Nonetheless, the bulk of 
         the evidence appears to uphold the serial hypothesis, at least for simple 
(20) objects that are relatively unfamiliar to the viewer.

Now look at the following question on a specific detail in the passage.

EXAMPLE

According to the passage, psychologists of the Gestalt school assume which of the following about the process of visual recognition?

  I. The image an object makes on the retina is exactly the same as its internal image.

 II. The mind recognizes a given object as a whole; it has no need to analyze the object’s constituent parts individually.

III. The process of matching an object with its internal image takes place in a single step.

(A) II only

(B) III only

(C) I and III only

(D) II and III only

(E) I, II, and III

You can arrive at the correct answer to this question by elimination.

First, quickly scan the passage looking for the key word Gestalt. The sentence mentioning Gestalt psychologists states they maintain that objects are recognized as wholes in a parallel procedure. The sentence immediately preceding defines a parallel procedure as one that takes only one step.

Now examine the statements. Do Gestalt psychologists maintain that an object’s retinal image is exactly the same as its internal image? Statement I is unsupported by the passage. Therefore, you can eliminate Choices C and E.

Statement II is supported by the passage: lines 11–12 indicate that Gestalt psychologists believe objects are recognized as wholes. Therefore, you can eliminate Choice B.

Statement III is supported by the passage: lines 12–13 indicate that Gestalt psychologists believe matching is a parallel process that occurs in one step. Therefore, you can eliminate Choice A.

Only Choice D is left. It is the correct answer.

Note how necessary it is to point to specific lines in the passage when you answer questions on specific details.

 When Asked to Make Inferences, Base Your Answers on What the Passage Implies, Not What It States Directly

In Language in Thought and Action, S. I. Hayakawa defines an inference as “a statement about the unknown made on the basis of the known.”

Inference questions require you to use your own judgment. You must not take anything directly stated by the author as an inference. Instead, you must look for clues in the passage that you can use in deriving your own conclusion. You should choose as your answer a statement that is a logical development of the information the author has provided.

Try this relatively easy inference question, based on the previous passage about visual recognition.

EXAMPLE

One can infer from the passage that, in visual recognition, the process of matching

(A) requires neural inactivity

(B) cannot take place if an attribute of a familiar object has been altered in some way

(C) cannot occur when the observer looks at an object for the very first time

(D) has now been proven to necessitate both serial and parallel processes

(E) can only occur when the brain receives a retinal image as a single unit

Go through the answer choices, eliminating any choices that obviously contradict what the passage states or implies. Remember that in answering inference questions you must go beyond the obvious, beyond what the authors explicitly state, to look for logical implications of what they say.

Choice A is incorrect. Nothing in the passage suggests that the matching process requires or demands neural inactivity. Rather, the entire process of visual recognition, including the matching of images, requires neural activity.

Choice D is incorrect. It is clear from the passage that the matching process is not fully understood; nothing yet has been absolutely proven. The weight of the evidence seems to support the serial hypothesis, but controversy still surrounds the entire question.

Choice E is incorrect. It can be eliminated because it directly contradicts information in the passage stating that recognition most likely is a serial or step-by-step process rather than a parallel one receiving an image as a single unit.

Choices B and C are left. Which is a possible inference? Choice C seems a possible inference. Although the author never says so, it seems logical that you could not match an object if you had never seen it before. After all, if you had never seen the object before, you would have no prior internal image of it and would have nothing with which to match it. What of Choice B? Nothing in the passage mentions altering any attributes or features of a familiar object. Therefore, on the basis of the passage you have no way to deduce whether matching would or would not be possible if such a change took place. There is not enough information in the passage to justify Choice B as an inference. The correct answer is Choice C.

Another, more difficult inference question is based on the previous excerpt reviewing Wilson’s Sociobiology. Review the passage briefly and see how you do with a question that very few of the examinees would have answered correctly.

         According to Wilson, only when we are able to apply the same parameters 
         and mathematical principles to weighing both troops of rhesus 
         macaques and termite colonies will a unified science of sociobiology 
Line finally exist. While recognizing that many of his colleagues question such 
  (5) an outcome, Wilson, one of sociobiology’s leading proponents, finds himself 
        simultaneously more and more struck by the functional similarities 
         that characterize both insect and vertebrate societies and less concerned 
         with the structural differences that divide them to such an apparently 
         irreconcilable degree. Thus, he freely compares termites and macaques, 
 (10) pointing out numerous likenesses between them. Both societies are territorial: 
        they occupy a particular home range, which they defend against 
         intruders. Likewise, both are cooperative: members organize themselves 
         into working groups that observe a clearly-defined division of labor. In 
         addition, members of both groups can convey to each other a range of 
 (15) basic emotions and personal information: animosity, fright, hunger, rank 
         within a particular caste, and ability to reproduce. Wilson readily 
         concedes that, from a specialist’s perspective, such a likeness may at first 
         appear superficial, even unscientifically glib. Nonetheless, in this eminent 
         scholar’s judgment, “it is out of such deliberate oversimplification that 
 (20) the beginnings of a general theory are made.”

EXAMPLE

In analyzing insect and vertebrate societies, the passage suggests which of the following?

(A) A clearly-defined division of labor is a distinguishing feature of most insect and vertebrate societies.

(B) The caste structures of insect and vertebrate societies share certain likenesses.

(C) Most insect and vertebrate societies utilize cooperative groups to hold and defend their home range.

(D) The system of communication employed by members of insect societies resembles the system that members of vertebrate societies follow.

(E) Major structural differences exist between insect and vertebrate societies.

Why would most examinees answer this question incorrectly? The reason is simple: it is easy to confuse statements made about specific insect and vertebrate societies with statements made about insect and vertebrate societies in general. In this passage, in the fourth sentence, the author switches from talking about Wilson’s views of insect and vertebrate societies in general and refers to his comments on termites and macaques in specific.

Go through the answer choices one by one. Does the passage suggest that a clearly-defined division of labor distinguishes most insect and vertebrate societies? No. It merely states that, according to Wilson, a clearcut division of labor is a characteristic of termite and rhesus macaque societies. Choice A is incorrect: you cannot justify leaping from a single type of insect (termites) and a single type of vertebrate (rhesus macaques) to most insects and most vertebrates.

Does the passage suggest that the caste structure of insect societies shares certain likenesses with that of their counterparts in vertebrate societies? No. It merely states that, according to Wilson, termites and macaques both can communicate rank within a particular caste. Choice B is incorrect. You cannot assume that the caste structure of insect societies is similar to the caste structure of vertebrate societies just because termites and rhesus macaques both have some way to communicate caste status or rank.

Does the passage suggest that most insect and vertebrate societies form cooperative groups in order to hold and defend their home range or territory? No. It merely states that termites and macaques organize themselves into cooperative groups, and that both species occupy and defend territories. Choice C is incorrect: again, you cannot justify leaping from termites and rhesus macaques to most insects and most vertebrates.

Does the passage suggest that the system of communication employed by members of insect societies resembles that employed by members of vertebrate societies? No. It merely states that communication among termites and macaques serves similar ends; it says nothing about the specific systems of communication they use, nor about those systems of communication used by other insects and vertebrates. Choice D is incorrect.

The correct answer is Choice E. In the passage, the author states that Wilson has grown less impressed “with the structural differences that divide them (i.e., insect and vertebrate societies) to such an apparently irreconcilable degree.” This suggests that, even though Wilson may be unimpressed with them, these differences exist and are major.

 When Asked to Apply Ideas from the Passage to a New Situation, Put Yourself in the Author’s Place

GRE application questions require you to do three things:

1.            Reason — If X is true, then Y must also be true.

2.            Perceive Feelings — If the author feels this way about subject A, he probably feels a certain way about subject B.

3.            Sense a Larger Structure — This passage is part of an argument for a proposal, or part of a description of a process, or part of a critique of a hypothesis.

Like inference questions, application questions require you to go beyond what the author explicitly states. Application questions, however, ask you to go well beyond a simple inference, using clues in the passage to interpret possible reasons for actions and possible outcomes of events. Your concern is to comprehend how the author’s ideas might apply to other situations, or be affected by them. To do so, you have to put yourself in the author’s place.

Imagine you are the author. What are you arguing for? Given what you have just stated in the passage, what would you want to say next? What might hurt your argument? What might make it stronger? What kind of audience would appreciate what you have to say? Whom are you trying to convince? If you involve yourself personally with the passage, you will be better able to grasp it in its entirety and see its significance.

Answer the following application question based on the previous passage discussing Wilson’s Sociobiology.

EXAMPLE

Which of the following statements would be most likely to begin the paragraph immediately following the passage?

(A) Wilson has raised a problem in ethical philosophy in order to characterize the essence of the discipline of sociobiology.

(B) It may not be too much to say that sociology and the other social sciences are the last branches of biology waiting to be integrated into neo-Darwinist evolutionary theory.

(C) Although behavioral biology is traditionally spoken of as if it were a unified subject, it is now emerging as two distinct disciplines centered on neurophysiology and sociobiology, respectively.

(D) The formulation of a theory of sociobiology constitutes, in Wilson’s opinion, one of the great manageable problems of biology for the next twenty or thirty years.

(E) In the past, the development of sociobiology has been slowed by too close an identification with ethology and behavioral psychology.

As you know from answering the previous main idea and attitude questions, Wilson’s point is that students of insect and vertebrate societies may be on the verge of devising a general theory of sociobiology. Like Wilson, the author of the passage appears optimistic about the likelihood of developing this unified science. At the same time, again like Wilson, he is cautious; he too does not wish to overstate the case.

Put yourself in the author’s place. What would you be likely to say next? The author has just been describing Wilson’s hopeful view of the prospects for putting together a general theory of sociobiology. What would be more natural than for him next to discuss Wilson’s opinion of a time frame for formulating this general theory? Choice D, with its confident yet judicious view of the formulation of a theory of sociobiology as “one of the great manageable problems of biology for the next twenty or thirty years,” seems a logical extension of what the passage has just been saying. While Choices A, B, C, and E all touch on sociobiology in some way, none of them follows as naturally from the passage’s immediate argument.

 When Asked to Give the Meaning of an Unfamiliar Word, Look for Nearby Context Clues

When a question in the reading comprehension part of an examination asks for the meaning of a word, that meaning can usually be deduced from the word’s context. The purpose of this kind of question is to determine how well you can extract meaning from the text, not how extensive your general vocabulary is.

Sometimes the unknown word is a common word used in one of its special or technical meanings. For example:

He threw the pot in an hour. The wheel turned busily and the shape grew quickly as his fingers worked the wet, spinning clay. (Throw here means to shape on a potter’s wheel.)

At other times, the unknown word may bear a deceptive resemblance to a known word.

He fell senseless to the ground. (He was unconscious. He did not fall foolishly or nonsensically to the ground.)

Just because you know one meaning of a word, do not assume that you know its meaning as it is used in a particular passage. You must look within the passage for clues. Often authors will use an unfamiliar word and then immediately define it within the same sentence. The two words or groups of words are juxtaposed — set beside one another — to make their relationship clear. Commas, hyphens, and parentheses may signal this relationship.

1.            The rebec, a medieval stringed instrument played with a bow, has only three strings.

2.            Paleontologists — students of fossil remains — explore the earth’s history.

3.            Most mammals are quadrupeds (four-footed animals).

Often an unfamiliar word in one clause of a sentence will be defined or clarified in the sentence’s other clause.

1.            The early morning dew had frozen, and everything was covered with a thin coat of rime.

2.            Cowards, we use euphemisms when we cannot bear the truth, calling our dead “the dear departed,” as if they have just left the room.

Refer once more to the passage on visual recognition to answer the following question.

         What is involved in the process of visual recognition? First, like computer 
         data, visual memories of an object must be stored; then, a mechanism 
         must exist for them to be retrieved. But how does this process work? 
Line The eye triggers the nerves into action. This neural activity constructs a 
  (5) picture in the brain’s memory system, an internal image of the object 
         observed. When the eye once again confronts that object, the object is 
         compared with its internal image; if the two images match, recognition 
         takes place.

         Among psychologists, the question as to whether visual recognition is 
 (10) a parallel, single-step operation or a sequential, step-by-step one is the 
         subject of much debate. Gestalt psychologists contend that objects are 
         perceived as wholes in a parallel operation: the internal image is matched 
         with the retinal impression in one single step. Psychologists of other 
         schools, however, suggest the opposite, maintaining that the individual 
 (15) features of an object are matched serially with the features of its internal 
         image. Some experiments have demonstrated that the more wellknown 
         an object is, the more holistic its internal image becomes, and the more 
         parallel the process of recognition tends to be. Nonetheless, the bulk of 
         the evidence appears to uphold the serial hypothesis, at least for simple 
 (20) objects that are relatively unfamiliar to the viewer.

EXAMPLE

Which of the following phrases could best replace “the more holistic its internal image becomes” (line 17) without significantly changing the sentence’s meaning?

(A) the more its internal image increases in detail

(B) the more integrated its internal image grows

(C) the more its internal image decreases in size

(D) the more it reflects its internal image

(E) the more indistinct its internal image appears

What words or phrases in the vicinity of “the more holistic its internal image becomes” give you a clue to the phrase’s meaning? The phrase immediately following, “becomes more parallel.” If the recognition process becomes more parallel as an object becomes more familiar, then matching takes place in one step in which all the object’s features are simultaneously transformed into a single internal representation. Thus, to say that an object’s internal image becomes more holistic is to say that it becomes more integrated or whole. The correct answer is Choice B.

 Familiarize Yourself with the Technical Terms Used to Describe a Passage’s Organization

Another aspect of understanding the author’s point is understanding how the author organizes what he has to say. You have to understand how the author makes his point, figure out whether he begins with his thesis or main idea or works up to it gradually. Often this means observing how the opening sentence or paragraph relates to the passage as a whole.

Here is a technique question based on the last two sentences of the passage about sociobiology. Those lines are repeated here so that you can easily refer to them.

Wilson readily concedes that, from a specialist’s perspective, such a likeness may at first appear superficial, even unscientifically glib. Nonetheless, in this eminent scholar’s judgment, “it is out of such deliberate oversimplification that the beginnings of a general theory are made.”

EXAMPLE

Which of the following statements best describes the organization of the author’s discussion of the importance of the termite/macaque comparison in the development of a unified science of sociobiology (lines 16–20)?

(A) He provides an example of a comparison and then rejects its implications.

(B) He concedes that current data are insufficient and modifies his initial assertion of their importance.

(C) He acknowledges hypothetical objections to the comparison, but concludes by reaffirming its significance.

(D) He cites critical appraisals of the comparison, but refrains from making an appraisal of his own.

(E) He notes an ambiguity in the comparison, but finally concedes its validity.

Consider the first clause of each answer choice.

In his comment on how things may seem from the specialist’s point of view, does the author provide an example of a comparison? No. He refers to a comparison made earlier. Therefore, you can eliminate Choice A.

Does he concede the insufficiency of current data? Not quite. He states that some people may quarrel with the comparison because it seems glib to them; he does not grant that they are right or that the data are inadequate. Therefore, you can eliminate Choice B.

Does he acknowledge hypothetical objections to the comparison? Definitely. Make a note to come back later to Choice C.

Does he cite critical appraisals of the comparison? Possibly. Again, make a note of Choice D.

Does he note an ambiguity in the comparison? No. He notes an objection to the comparison; he mentions no ambiguities within it. Therefore, you can eliminate Choice E.

Now consider the second clause of Choices C and D. Does the author refrain from making an appraisal of the comparison? No. He calls it a deliberate oversimplification that may bear fruit. Choice D is incorrect. Does the author conclude by reaffirming the significance of the termite/macaque comparison? Clearly he does; he quotes Wilson’s conclusion that such oversimplified comparisons can provide the basis for an important general theory. The correct answer is Choice C.

Practice Exercises

Note: Although the reading passages on the computer-based GRE range from 100 to 400 words in length, the paper-based GRE taken by students in foreign countries includes reading passages of up to 800 words in length. Therefore, the following practice exercises present a selection of long and short passages to help students to prepare for either the computer-based or the paper-based test.

Directions: Each of the following reading comprehension questions are based on the content of the following passage. Read the passage and then determine the best answer choice for each question. Base your choice on what this passage states directly or implies, not on any information you may have gained elsewhere.

Reading Comprehension Exercise A

         One phase of the business cycle is the 
         expansion phase. This phase is a twofold one, 
         including recovery and prosperity. During the 
Line recovery period there is ever-growing expansion 
  (5) of existing facilities, and new facilities for 
         production are created. More businesses are 
         created and older ones expanded. Improvements 
         of various kinds are made. There is an 
         ever-increasing optimism about the future of 
 (10) economic growth. Much capital is invested in 
         machinery or “heavy” industry. More labor is 
         employed. More raw materials are required. 
         As one part of the economy develops, other 
         parts are affected. For example, a great expansion 
(15) in automobiles results in an expansion of 
         the steel, glass, and rubber industries. Roads 
         are required; thus the cement and machinery 
         industries are stimulated. Demand for labor 
         and materials results in greater prosperity for 
 (20) workers and suppliers of raw materials, 
         including farmers. This increases purchasing 
         power and the volume of goods bought and 
         sold. Thus prosperity is diffused among the 
         various segments of the population. This 
 (25) prosperity period may continue to 
         rise and rise without an apparent end. However, a 
         time comes when this phase reaches a peak 
         and stops spiralling upwards. This is the end 
         of the expansion phase.

1. Which of the following statements is the best example of the optimism mentioned in line 9 of the passage as being part of the expansion phase?

(A) Public funds are designated for the construction of new highways designed to stimulate tourism.

(B) Industrial firms allocate monies for the purchase of machine tools.

(C) The prices of agricultural commodities are increased at the producer level.

(D) Full employment is achieved at all levels of the economy.

(E) As technology advances, innovative businesses replace antiquated firms.

2. It can be inferred from the passage that the author believes that

(A) when consumers lose their confidence in the market, a recession follows

(B) cyclical ends to business expansion are normal

(C) luxury goods such as jewelry are unaffected by industrial expansion

(D) with sound economic policies, prosperity can become a fixed pattern

(E) the creation of new products is essential for prosperity

3. Which of the following statements would be most likely to begin the paragraph immediately following the passage?

(A) Union demands may also have an effect on business cycles.

(B) Some industries are, by their very nature, cyclical, having regular phases of expansion and recession.

(C) Inflation is a factor that must be taken into consideration in any discussion of the expansion phase.

(D) The farmer’s role during the expansion phase is of vital importance.

(E) The other phase of the business cycle is called the recession phase.

         The history of mammals dates back at least 
         to Triassic time. Development was retarded, 
         however, until the sudden acceleration of evolutional 
Line change that occurred in the oldest 
  (5) Paleocene. This led in Eocene time to increase 
         in average size, larger mental capacity, and 
         special adaptations for different modes of life. 
         In the Oligocene Epoch, there was further 
         improvement, with appearance of some new 
 (10) lines and extinction of others. Miocene and 
         Pliocene time was marked by culmination of 
         several groups and continued approach 
         toward modern characters. The peak of the 
         career of mammals in variety and average 
 (15) large size was attained in the Miocene.

         The adaptation of mammals to almost all 
         possible modes of life parallels that of the reptiles 
         in Mesozoic time, and except for greater 
         intelligence, the mammals do not seem to 
 (20) have done much better than corresponding 
         reptilian forms. The bat is doubtless a better 
         flying animal than the pterosaur, but the dolphin 
         and whale are hardly more fishlike than 
         the ichthyosaur. Many swift-running 
 (25) mammals of the plains, like the horse and the antelope, 
         must excel any of the dinosaurs. The 
         tyrannosaur was a more ponderous and powerful 
         carnivore than any flesh-eating mammal, 
         but the lion or tiger is probably a more 
 (30) efficient and dangerous beast of prey because of a 
         superior brain. The significant point to 
         observe is that different branches of the 
         mammals gradually fitted themselves for all sorts of 
         life, grazing on the plains and able to run 
 (35) swiftly (horse, deer, bison), living in rivers and 
         swamps (hippopotamus, beaver), dwelling in 
         trees (sloth, monkey), digging underground 
         (mole, rodent), feeding on flesh in the forest 
         (tiger) and on the plain (wolf), swimming in 
 (40) the sea (dolphin, whale, seal), and flying in the 
         air (bat). Man is able by mechanical means to 
         conquer the physical world and to adapt himself 
         to almost any set of conditions.

         This adaptation produces gradual changes 
 (45) of form and structure. It is biologically characteristic 
         of the youthful, plastic stage of a group. 
         Early in its career, an animal assemblage seems 
         to possess capacity for change, which, as the 
         unit becomes old and fixed, disappears. The 
 (50) generalized types of organisms retain longest 
         the ability to make adjustments when 
         required, and it is from them that new, fecund 
         stocks take origin — certainly not from any 
         specialized end products. So, in the mammals, 
 (55) we witness the birth, plastic spread in many 
         directions, increasing specialization, and in 
         some branches, the extinction, which we have 
         learned from observation of the geologic record 
         of life is a characteristic of the evolution of life.

4. Which of the following would be the most appropriate title for the passage?

(A) From Dinosaur to Man

(B) Adaptation and Extinction

(C) The Superiority of Mammals

(D) The Geologic Life Span

(E) Man, Conqueror of the Physical World

5. It can be inferred from the passage that the chronological order of the geologic periods is

(A) Paleocene, Miocene, Triassic, Mesozoic

(B) Paleocene, Triassic, Mesozoic, Miocene

(C) Miocene, Paleocene, Triassic, Mesozoic

(D) Mesozoic, Oligocene, Paleocene, Miocene

(E) Mesozoic, Paleocene, Eocene, Miocene

6. It can be inferred from the passage that the pterosaur

(A) resembled the bat

(B) was a Mesozoic mammal

(C) was a flying reptile

(D) lived in the sea

(E) evolved during the Miocene period

7. According to the passage, the greatest number of forms of mammalian life is found in the

(A) Triassic period

(B) Eocene period

(C) Oligocene period

(D) Pliocene period

(E) Miocene period

8. Which of the following statements, if true, would weaken the statement made by the author in lines 16–21?

(A) Tyrannosaur has been found to have a larger brain than was previously thought.

(B) Mammals will become extinct within the next thousand years.

(C) Forms of flying ichthyosaurs have recently been discovered.

(D) The tiger has now been proved to be more powerful than the carnivorous reptiles.

(E) Computers have been developed that can double human mental capacity.

9. It can be inferred from the passage that the evidence the author uses in discussing the life of past time periods

(A) was developed by Charles Darwin

(B) was uncovered by the author

(C) has been negated by more recent evidence

(D) was never definitely established

(E) is based on fossil remains

10. With which of the following proverbial expressions about human existence would the author be most likely to agree?

(A) It’s a cruel world.

(B) All the world’s a stage.

(C) The more things change, the more they remain the same.

(D) Footprints in the sands of time.

(E) A short life, but a merry one.

         For me, scientific knowledge is divided into 
         mathematical natural sciences or sciences 
         dealing with the natural world (physical 
Line and biological sciences), and sciences dealing 
  (5) with mankind (psychology, sociology, all the 
         sciences of cultural achievements, every kind of 
         historical knowledge). Apart from these sciences 
         is philosophy, about which we will talk 
         shortly. In the first place, all this is pure or 
 (10) theoretical knowledge, sought only for the 
         purpose of understanding, in order to fulfill the 
         need to understand that is intrinsic and consubstantial 
         to man. What distinguishes man 
         from animal is that he knows and needs to 
 (15) know. If man did not know that the world 
         existed, and that the world was of a certain 
         kind, that he was in the world and that he himself 
         was of a certain kind, he wouldn’t be man. 
         The technical aspects of applications of knowledge 
 (20) are equally necessary for man and are of 
         the greatest importance, because they also contribute 
         to defining him as man and permit him 
         to pursue a life increasingly more truly human.

         But even while enjoying the results of technical 
 (25) progress, he must defend the primacy and 
         autonomy of pure knowledge. Knowledge 
         sought directly for its practical applications will 
         have immediate and foreseeable success, but not 
         the kind of important result whose revolutionary 
 (30) scope is in large part unforeseen, except by 
         the imagination of the Utopians. Let me recall 
         a well-known example. If the Greek mathematicians 
         had not applied themselves to the investigation 
         of conic sections, zealously and without 
 (35) the least suspicion that it might someday be 
         useful, it would not have been possible centuries 
         later to navigate far from shore. The first 
         men to study the nature of electricity could not 
         imagine that their experiments, carried on 
 (40) because of mere intellectual curiosity, would 
         eventually lead to modern electrical technology, 
         without which we can scarcely conceive of contemporary 
         life. Pure knowledge is valuable for 
         its own sake, because the human spirit cannot 
 (45) resign itself to ignorance. But, in addition, it is 
         the foundation for practical results that would 
         not have been reached if this knowledge had 
         not been sought disinterestedly.

11. The author points out that the Greeks who studied conic sections

(A) invented modern mathematical applications

(B) were interested in navigation

(C) were unaware of the value of their studies

(D) worked with electricity

(E) were forced to resign themselves to failure

12. The title below that best expresses the ideas of this passage is

(A) Technical Progress

(B) A Little Learning Is a Dangerous Thing

(C) Man’s Distinguishing Characteristics

(D) Learning for Its Own Sake

(E) The Difference Between Science and Philosophy

13. It can be inferred from the passage that to the author man’s need to know is chiefly important in that it

(A) allows the human race to progress technically

(B) encompasses both the physical and social sciences

(C) demonstrates human vulnerability

(D) defines man’s essential humanity

(E) has increased as our knowledge of the world has grown

         When you first saw a piece of African art, it 
         impressed you as a unit; you did not see it as a 
         collection of shapes or forms. This, of course, 
Line means that the shapes and volumes within the 
  (5) sculpture itself were coordinated so successfully 
         that the viewer was affected emotionally. 
         It is entirely valid to ask how, from a purely 
         artistic point of view, this unity was achieved. 
         And we must also inquire whether there is a 
 (10) recurrent pattern or rules or a plastic language 
         and vocabulary which is responsible for the 
         powerful communication of emotion which 
         the best African sculpture achieves. If there is 
         such a pattern or rules, are these rules applied 
 (15) consciously or instinctively to obtain so many 
         works of such high artistic quality?

         It is obvious from the study of art history 
         that an intense and unified emotional experience, 
         such as the Christian credo of the 
 (20) Byzantine or 12th or 13th century Europe, 
         when expressed in art forms, gave great unity, 
         coherence, and power to art. But such an integrated 
         feeling was only the inspirational 
         element for the artist, only the starting point of 
 (25) the creative act. The expression of this emotion 
         and its realization in the work could be done 
         only with discipline and thorough knowledge 
         of the craft. And the African sculptor was a 
         highly trained workman. He started his apprenticeship 
 (30) with a master when a child, and he 
         learned the tribal styles and the use of tools and 
         the nature of woods so thoroughly that his carving 
         became what Boas calls “motor action.” He 
         carved automatically and instinctively. 
(35)         The African carver followed his rules without 
         thinking of them; indeed, they never seem 
         to have been formulated in words. But such 
         rules existed, for accident and coincidence 
         cannot explain the common plastic language 
 (40) of African sculpture. There is too great a consistency 
         from one work to another. Yet, 
         although the African, with amazing insight 
         into art, used these rules, I am certain that he 
         was not conscious of them. This is the great 
 (45) mystery of such a traditional art: talent, or the 
         ability certain people have, without conscious 
         effort, to follow the rules which later the analyst 
         can discover only from the work of art 
         which has already been created.

14. The author is primarily concerned with

(A) discussing how African sculptors achieved their effects

(B) listing the rules followed in African art

(C) relating African art to the art of 12th or 13th century Europe

(D) integrating emotion and realization

(E) expressing the beauty of African art

15. According to the passage, one of the outstanding features of African sculpture is

(A) its esoteric subject matter

(B) the emotional content of the work

(C) the education or training of the artists

(D) its “foreignness” when compared to Western art

(E) its high degree of conscious control

16. The author uses the phrase “plastic language” in lines 10 and 39 to refer to African art’s

(A) mass reproduction

(B) unrealistic qualities

(C) modernistic orientation

(D) sculptural symbols

(E) repetitive nature

17. The information in the passage suggests that an African carver might best be compared to a

(A) chef following a recipe

(B) fluent speaker of English who is just beginning to study French

(C) batter who hits a home run in his or her first baseball game

(D) concert pianist performing a well-rehearsed concerto

(E) writer who is grammatically expert but stylistically uncreative

18. Which of the following does the passage imply about art?

(A) Content is more important than form.

(B) There is no room for untrained artists.

(C) Form is more important than content.

(D) Western artists are too concerned with technique.

(E) Great art must be consistent.

19. The author’s presentation of the material includes all of the following EXCEPT

(A) comparison

(B) cause and effect

(C) rhetorical questioning

(D) direct quotation

(E) concrete example

20. Which of the following titles best expresses the content of the passage?

(A) The Apprenticeship of the African Sculptor

(B) The History of African Sculpture

(C) How African Art Achieves Unity

(D) Analyzing African Art

(E) The Unconscious Rules of African Art

Reading Comprehension Exercise B

         Both plants and animals of many sorts 
         show remarkable changes in form, structure, 
         growth habits, and even mode of reproduction 
Line in becoming adapted to different climatic 
  (5) environment, types of food supply, or mode 
         of living. This divergence in response to evolution 
         is commonly expressed by altering the 
         form and function of some part or parts of the 
         organism, the original identity of which is 
 (10) clearly discernible. For example, the creeping 
         foot of the snail is seen in related marine 
         pteropods to be modified into a flapping 
         organ useful for swimming, and is changed 
         into prehensile arms that bear suctorial disks in 
 (15) the squids and other cephalopods. The limbs 
         of various mammals are modified according 
         to several different modes of life — for swift 
         running (cursorial) as in the horse and antelope, 
         for swinging in trees (arboreal) as in the 
 (20) monkeys, for digging (fossorial) as in the 
         moles and gophers, for flying (volant) as in 
         the bats, for swimming (aquatic) as in the 
         seals, whales, and dolphins, and for other 
         adaptations. The structures or organs that 
 (25) show main change in connection with this 
         adaptive divergence are commonly identified 
         readily as homologous, in spite of great alterations. 
         Thus, the finger and wristbones of a bat 
         and whale, for instance, have virtually nothing 
 (30) in common except that they are definitely 
         equivalent elements of the mammalian limb.

1. Which of the following is the most appropriate title for the passage, based on its content?

(A) Adaptive Divergence

(B) Evolution

(C) Unusual Structures

(D) Changes in Organs

(E) Our Changing Bodies

2. The author provides information that would answer which of the following questions?

  I. What factors cause change in organisms?

 II. What is the theory of evolution?

III. How are horses’ legs related to seals’ flippers?

(A) I only

(B) II only

(C) I and II only

(D) I and III only

(E) I, II, and III

3. Which of the following words could best be substituted for “homologous” (line 27) without substantially changing the author’s meaning?

(A) altered

(B) mammalian

(C) corresponding

(D) divergent

(E) tactile

4. The author’s style can best be described as

(A) humorous

(B) objective

(C) patronizing

(D) esoteric

(E) archaic

         Plato — who may have understood better 
         what forms the mind of man than do some of 
         our contemporaries who want their children 
Line exposed only to “real” people and everyday 
  (5) events — knew what intellectual experiences 
         make for true humanity. He suggested that 
         the future citizens of his ideal republic begin 
         their literary education with the telling of 
         myths, rather than with mere facts or so-called 
 (10) rational teachings. Even Aristotle, master 
         of pure reason, said: “The friend of 
         wisdom is also a friend of myth.”

         Modern thinkers who have studied myths 
         and fairy tales from a philosophical or psychological 
 (15) viewpoint arrive at the same conclusion, 
         regardless of their original persuasion. 
         Mircea Eliade, for one, describes these stories 
         as “models for human behavior [that], by that 
         very fact, give meaning and value to life.” 
 (20) Drawing on anthro pological parallels, he and 
         others suggest that myths and fairy tales were 
         derived from, or give symbolic expression to, 
         initiation rites or rites of passage — such as 
         metaphoric death of an old, inadequate self in 
 (25) order to be reborn on a higher plane of existence. 
         He feels that this is why these tales 
         meet a strongly felt need and are carriers of 
         such deep meaning.

         Other investigators with a depth-psychological 
 (30) orientation emphasize the similarities 
         between the fantastic events in myths and 
         fairy tales and those in adult dreams and daydreams 
         — the fulfillment of wishes, the winning 
         out over all competitors, the destruction 
 (35) of enemies — and conclude that one attraction 
         of this literature is its expression of that 
         which is normally prevented from coming to 
         awareness.

         There are, of course, very significant differences 
 (40) between fairy tales and dreams. For 
         example, in dreams more often than not the 
         wish fulfillment is disguised, while in fairy 
         tales much of it is openly expressed. To a considerable 
         degree, dreams are the result of inner 
 (45) pressures which have found no relief, of problems 
         which beset a person to which he knows 
         no solution and to which the dream finds 
         none. The fairy tale does the opposite: it projects 
         the relief of all pressures and not only 
 (50) offers ways to solve problems but promises 
         that a “happy” solution will be found.

         We cannot control what goes on in our 
         dreams. Although our inner censorship influences 
         what we may dream, such control 
 (55) occurs on an unconscious level. The fairy tale, 
         on the other hand, is very much the result of 
         common conscious and unconscious content 
         having been shaped by the conscious mind, 
         not of one particular person, but the consensus 
 (60) of many in regard to what they view as 
         universal human problems, and what they 
         accept as desirable solutions. If all these elements 
         were not present in a fairy tale, it would 
         not be retold by generation after generation. 
 (65) Only if a fairy tale met the conscious and 
         unconscious requirements of many people 
         was it repeatedly retold, and listened to with 
         great interest. No dream of a person could 
         arouse such persistent interest unless it was 
 (70) worked into a myth, as was the story of the 
         pharaoh’s dream as interpreted by Joseph in 
         the Bible.

5. It can be inferred from the passage that the author’s interest in fairy tales centers chiefly on their

(A) literary qualities

(B) historical background

(C) factual accuracy

(D) psychological relevance

(E) ethical weakness

6. According to the passage, fairy tales differ from dreams in which of the following characteristics?

  I. The communal nature of their creation

 II. Their convention of a happy ending

III. Their enduring general appeal

(A) I only

(B) II only

(C) I and II only

(D) II and III only

(E) I, II, and III

7. It can be inferred from the passage that Mircea Eliade is most likely

(A) a writer of children’s literature

(B) a student of physical anthropology

(C) a twentieth-century philosopher

(D) an advocate of practical education

(E) a contemporary of Plato

8. Which of the following best describes the author’s attitude toward fairy tales?

(A) Reluctant fascination

(B) Wary skepticism

(C) Scornful disapprobation

(D) Indulgent tolerance

(E) Open approval

9. The author cites Plato and Aristotle primarily in order to

(A) define the nature of myth

(B) contrast their opposing points of view

(C) support the point that myths are valuable

(D) prove that myths originated in ancient times

(E) give an example of depth psychology

10. The author mentions all of the following as reasons for reading fairy tales EXCEPT

(A) emotional catharsis

(B) behavioral paradigm

(C) uniqueness of experience

(D) sublimation of aggression

(E) symbolic satisfaction

         Nothing more unlucky, I sometimes think, 
         could have befallen Chaucer than that he 
         should have been christened “the father of 
Line English poetry.” For “father” in such a context 
  (5) conveys to most of us, I fear, a faint suggestion 
         of vicarious glory — the derivative celebrity of 
         parents, otherwise obscure, who shine, moonlike, 
         in the reflected luster of their sons. What 
         else than progenitors were the fathers of Plato, 
 (10) or Caesar, or Shakespeare, or Napoleon? And 
         so to call Chaucer the father of English poetry 
         is often tantamount to dismissing him, not 
         unkindly, as the estimable but archaic ancestor 
         of a brilliant line. But Chaucer — if I may 
 (15) risk the paradox — is himself the very thing 
         he begat. He is English poetry incarnate, and 
         only two, perhaps, of all his sons outshine his 
         fame. It is with Chaucer himself, then, and 
         not save incidentally with his ancestral eminence 
 (20) that we shall be concerned.

         But five hundred and thirty-three years have 
         passed since Chaucer died. And to overleap five 
         centuries is to find ourselves in another world, 
         a world at once familiar and strange. Its determining 
 (25) concepts are implicit in all that 
         Chaucer, who was of it, thought and wrote. 
         And, woven as they are into his web, they at 
         once lend to it and gain from it fresh significance. 
         To us they are obsolete; in the 
 (30) Canterbury Tales, and the Troilus, and the 
         House of Fame they are current and alive. And 
         it is in their habit as they lived, and not as mere 
         curious lore, that I mean to deal with them.

         Let me begin with the very tongue which 
 (35) Chaucer spoke — a speech at once our own 
         and not our own. “You know,” he wrote — 
         and for the moment I rudely modernize lines as 
         liquid in their rhythm as smooth-sliding 
         brandy — “you know that in a thousand years 
 (40) there is change in the forms of speech, and 
         words which were then judged apt and choice 
         now seem to us wondrous quaint and strange, 
         and yet they spoke them so, and managed as 
         well in love with them as men now do.” And to 
 (45) us, after only half a thousand years, those very 
         lines are an embodiment of what they state: 
            Ye knowe eek, that in forme of speche is 
         chaunge 
         With-inne a thousand yeer, and words tho 
 (50) That hadden prys, now wonder nyce and 
         straunge 
         Us thinketh hem; and yet they spake hem 
         so, 
         And spedde as wel in love as men now do. 
 (55) But it is not only Chaucer’s speech which 
         has undergone transformation. The change in 
         his world is greater still. And the situation 
         which confronts us is this. In Chaucer’s greatest 
         work we have to do with timeless creations 
 (60) upon a time-determined stage. And it is one of 
         the inescapable ironies of time that creations 
         of the imagination which are at once of no 
         time and for all time must nevertheless think 
         and speak and act in terms and in ways which 
 (65) are as transient as they themselves are permanent. 
         Their world — the stage on which they 
         play their parts, and in terms of which they 
         think — has become within a few lifetimes 
         strange and obsolete, and must be deciphered 
 (70) before it can be read. For the immortal puts 
         on mortality when great conceptions are 
         clothed in the only garment ever possible — 
         in terms whose import and associations are 
         fixed by the form and pressure of an inexorably 
 (75) passing time. And that is the situation 
         which we have to face.

11. The author of the passage does all of the following in the discussion of Chaucer and his verse EXCEPT

(A) pose a rhetorical question

(B) cite specific examples

(C) offer a personal opinion

(D) propose a solution

(E) use figurative language

12. The author’s attitude toward “mere curious lore” (line 32–33) can best be described as

(A) skeptical but resigned

(B) admiring and intrigued

(C) dismissive

(D) incredulous

(E) completely detached

13. The author uses the Middle English quotation (lines 47–54) to

(A) refute the contention that Chaucer wrote awkwardly

(B) demonstrate the idiosyncratic spelling common in Chaucer’s time

(C) convey the power of reading poetry in its original form

(D) support his hypothesis about the aptness of Chaucer’s choice of words

(E) illustrate the degree of linguistic change that has occurred

14. How would the author most likely respond to another critic’s use of the term “Father of English Poetry” to describe Chaucer?

(A) The term “Father of English Poetry” is an accurate assessment of an exceptionally distinguished literary figure.

(B) The term implies Chaucer is important not as a great poet in his own right but as the somewhat outdated forerunner of the great poets of today.

(C) The epithet “Father of English Poetry” has been applied to so many poets that it has lost whatever meaning it originally possessed.

(D) “Father of English Poetry” is a sexist term that should be replaced by more inclusive language.

(E) It is appropriate to acknowledge the impact Chaucer had on posterity by revering him as the glorious ancestor of all English poets.

         Of the 197 million square miles making up 
         the surface of the globe, 71 percent is covered 
         by the interconnecting bodies of marine 
Line water; the Pacific Ocean alone covers half the 
  (5) Earth and averages nearly 14,000 feet in 
         depth. The continents — Eurasia, Africa, 
         North America, South America, Australia, 
         and Antarctica — are the portions of the continental 
         masses rising above sea level. The submerged 
 (10) borders of the continental masses are 
         the continental shelves, beyond which lie the 
         deep-sea basins.

         The oceans attain their greatest depths not 
         in their central parts, but in certain elongated 
 (15) furrows, or long narrow troughs, called deeps
         These profound troughs have a peripheral 
         arrangement, notably around the borders of 
         the Pacific and Indian oceans. The position of 
         the deeps near the continental masses suggests 
 (20) that the deeps, like the highest mountains, are 
         of recent origin, since otherwise they would 
         have been filled with waste from the lands. 
         This suggestion is strengthened by the fact 
         that the deeps are frequently the sites of 
 (25) world-shaking earthquakes. For example, the 
         “tidal wave” that in April, 1946, caused widespread 
         destruction along Pacific coasts 
         resulted from a strong earthquake on the floor 
         of the Aleutian Deep. 
 (30)      The topography of the ocean floors is none 
         too well known, since in great areas the available 
         soundings are hundreds or even thousands 
         of miles apart. However, the floor of the 
         Atlantic is becoming fairly well known as a 
 (35) result of special surveys since 1920. A broad, 
         well-defined ridge — the Mid-Atlantic 
         ridge — runs north and south between Africa 
         and the two Americas, and numerous other 
         major irregularities diversify the Atlantic 
 (40) floor. Closely spaced soundings show that 
         many parts of the oceanic floors are as rugged 
         as mountainous regions of the continents. Use 
         of the recently perfected method of echo 
         sounding is rapidly enlarging our knowledge 
 (45) of submarine topography. During World War 
         II great strides were made in mapping submarine 
         surfaces, particularly in many parts of the 
         vast Pacific basin.

         The continents stand on the average 2870 
 (50) feet — slightly more than half a mile — above 
         sea level. North America averages 2300 feet; 
         Europe averages only 1150 feet; and Asia, the 
         highest of the larger continental subdivisions, 
         averages 3200 feet. The highest point on the 
 (55) globe, Mount Everest in the Himalayas, is 
         29,000 feet above the sea; and as the greatest 
         known depth in the sea is over 35,000 feet, the 
         maximum relief (that is, the difference in altitude 
         between the lowest and highest points) 
 (60) exceeds 64,000 feet, or exceeds 12 miles. The 
         continental masses and the deep-sea basins are 
         relief features of the first order; the deeps, 
         ridges, and volcanic cones that diversify the sea 
         floor, as well as the plains, plateaus, and 
 (65) mountains of the continents, are relief features 
         of the second order. The lands are unendingly 
         subject to a complex of activities summarized 
         in the term erosion, which first sculptures them 
         in great detail and then tends to reduce them 
 (70) ultimately to sea level. The modeling of the 
         landscape by weather, running water, and 
         other agents is apparent to the keenly observant 
         eye and causes thinking people to speculate 
         on what must be the final result of the 
 (75) ceaseless wearing down of the lands. Long 
         before there was a science of geology, 
         Shakespeare wrote “the revolution of the times 
         makes mountains level.”

15. Which of the following would be the most appropriate title for the passage?

(A) Features of the Earth’s Surface

(B) Marine Topography

(C) The Causes of Earthquakes

(D) Primary Geologic Considerations

(E) How to Prevent Erosion

16. It can be inferred from the passage that the largest ocean is the

(A) Atlantic

(B) Pacific

(C) Indian

(D) Antarctic

(E) Arctic

17. The “revolution of the times” as used in the final sentence means

(A) the passage of years

(B) the current rebellion

(C) the science of geology

(D) the action of the ocean floor

(E) the overthrow of natural forces

18. According to the passage, the peripheral furrows or deeps are found

(A) only in the Pacific and Indian oceans

(B) near earthquakes

(C) near the shore

(D) in the center of the ocean

(E) to be 14,000 feet in depth in the Pacific

19. The passage contains information that would answer which of the following questions?

  I. What is the highest point on North America?

 II. Which continental subdivision is, on the average, 1150 feet above sea level?

III. How deep is the deepest part of the ocean?

(A) I only

(B) II only

(C) III only

(D) I and II only

(E) II and III only

20. From this passage, it can be inferred that earthquakes

(A) occur only in the peripheral furrows

(B) occur more frequently in newly formed land or sea formations

(C) are a prime cause of soil erosion

(D) will ultimately “make mountains level”

(E) are caused by the weight of the water

Reading Comprehension Exercise C

         An essay which appeals chiefly to the intellect 
         is Francis Bacon’s “Of Studies.” His careful tripartite 
         division of studies expressed succinctly 
Line in aphoristic prose demands the complete 
  (5) attention of the mind of the reader. He considers 
         studies as they should be: for pleasure, for 
         self-improvement, for business. He considers 
         the evils of excess study: laziness, affectation, 
         and preciosity. Bacon divides books into three 
 (10) categories: those to be read in part, those to be 
         read cursorily, and those to be read with care. 
         Studies should include reading, which gives 
         depth; speaking, which adds readiness of 
         thought; and writing, which trains in preciseness. 
 (15) Somewhat mistakenly, the author ascribes 
         certain virtues to individual fields of study: wisdom 
         to history, wit to poetry, subtlety to mathematics, 
         and depth to natural philosophy. 
         Bacon’s four-hundred-word essay, studded with 
 (20) Latin phrases and highly compressed in 
         thought, has intellectual appeal indeed.

1. Which of the following is the most appropriate title for the passage, based on its content?

(A) Francis Bacon and the Appeal of the Essay

(B) “Of Studies”: A Tripartite Division

(C) An Intellectual Exercise: Francis Bacon’s “Of Studies”

(D) The Categorization of Books According to Bacon

(E) A Method for Reading Books

2. Which of the following words could best be substituted for “aphoristic” (line 4) without substantially changing the author’s meaning?

(A) abstruse

(B) pithy

(C) tripartite

(D) proverbial

(E) realistic

3. The passage suggests that the author would be most likely to agree with which of the following statements?

(A) “Of Studies” belongs in the category of works that demand to be read with care.

(B) Scholars’ personalities are shaped by the academic discipline in which they are engaged.

(C) It is an affectation to use foreign words in one’s writing.

(D) An author can be more persuasive in a long work than in a shorter one.

(E) Studies should be undertaken without thought of personal gain.

Rocks which have solidified directly from 
         molten materials are called igneous rocks. 
         Igneous rocks are commonly referred to as 
Line primary rocks because they are the original 
  (5) source of material found in sedimentaries and 
         metamorphics. Igneous rocks compose the 
         greater part of the earth’s crust, but they are 
         generally covered at the surface by a relatively 
         thin layer of sedimentary or metamorphic 
 (10) rocks. Igneous rocks are distinguished by the 
         following characteristics: (1) they contain no 
         fossils; (2) they have no regular arrangement 
         of layers; and (3) they are nearly always made 
         up of crystals. 
 (15)      Sedimentary rocks are composed largely of 
         minute fragments derived from the disintegration 
         of existing rocks and in some 
         instances from the remains of animals. As sediments 
         are transported, individual fragments 
 (20) are assorted according to size. Distinct layers 
         of such sediments as gravels, sand, and clay 
         build up, as they are deposited by water and 
         occasionally wind. These sediments vary in 
         size with the material and the power of the 
 (25) eroding agent. Sedimentary materials are laid 
         down in layers called strata.

         When sediments harden into sedimentary 
         rocks, the names applied to them change to 
         indicate the change in physical state. Thus, 
 (30) small stones and gravel cemented together are 
         known as conglomerates; cemented sand 
         becomes sandstone; and hardened clay 
         becomes shale. In addition to these, other sedimentary 
         rocks such as limestone frequently 
 (35) result from the deposition of dissolved material. 
         The ingredient parts are normally precipitated 
         by organic substances, such as shells of 
         clams or hard skeletons of other marine life.

         Both igneous and sedimentary rocks may be 
 (40) changed by pressure, heat, solution, or cementing 
         action. When individual grains from existing 
         rocks tend to deform and interlock, they 
         are called metamorphic rocks. For example, 
         granite, an igneous rock, may be metamorphosed 
 (45) into a gneiss or a schist. Limestone, a 
         sedimentary rock, when subjected to heat and 
         pressure may become marble, a metamorphic 
         rock. Shale under pressure becomes slate.

4. The primary purpose of the passage is to

(A) differentiate between and characterize igneous and sedimentary rocks

(B) explain the factors that may cause rocks to change in form

(C) show how the scientific names of rocks reflect the rocks’ composition

(D) define and describe several diverse kinds of rocks

(E) explain why rocks are basic parts of the earth’s structure

5. All of the following are sedimentary rocks EXCEPT

(A) shale

(B) gravel

(C) sand

(D) limestone

(E) schist

6. The passage would be most likely to appear in a

(A) technical article for geologists

(B) teaching manual accompanying an earth science text

(C) pamphlet promoting conservation of natural resources

(D) newspaper feature explaining how oil is found

(E) nonfiction book explaining where to find the results of sedimentation

7. The relationship between igneous and sedimentary rocks may best be compared to the relationship between

(A) leaves and compost

(B) water and land

(C) DNA and heredity

(D) nucleus and cell wall

(E) sand and clay

8. The passage contains information that would answer which of the following questions?

  I. Which elements form igneous rocks?

 II. What produces sufficient pressure to alter a rock?

III. Why is marble called a metamorphic rock?

(A) I only

(B) III only

(C) I and II only

(D) II and III only

(E) I, II, and III

9. Which of the following methods is NOT used by the author?

(A) inclusion of concrete examples

(B) classification and discussion

(C) comparison and contrast

(D) observation and hypothesis

(E) cause and effect

10. The author’s tone in the passage can best be described as

(A) meditative

(B) objective

(C) ironic

(D) concerned

(E) bombastic

         Although vocal cords are lacking in 
         cetaceans, phonation is undoubtedly centered 
         in the larynx. 
Line      The toothed whales or odontocetes (sperm 
  (5) whale and porpoises) are much more vociferous 
         than the whalebone whales, or mysticetes. In 
         this country observers have recorded only occasional 
         sounds from two species of mysticetes 
         (the humpback and right whale). A Russian 
 (10) cetologist reports hearing sounds from at least 
         five species of whalebone whales but gives no 
         details of the circumstances or descriptions of 
         the sounds themselves. Although comparison 
         of the sound-producing apparatus in the two 
 (15) whale groups cannot yet be made, it is interesting 
         to note that the auditory centers of the 
         brain are much more highly developed in the 
         odontocetes than in the mysticetes, in fact, to a 
         degree unsurpassed by any other mammalian 
 (20) group.

11. The passage contains information that would answer which of the following questions?

  I. What are odontocetes and mysticetes?

 II. In which part of the body do whales produce sounds?

III. In which animals is the auditory center of the brain most developed?

(A) I only

(B) II only

(C) I and II only

(D) II and III only

(E) I, II, and III

12. The author’s attitude toward the observations reported by the Russian cetologist mentioned in lines 9–13 is best described as one of

(A) admiration

(B) indignation

(C) surprise

(D) skepticism

(E) pessimism

13. It can be inferred from the passage that

(A) animals with more highly developed auditory apparatuses tend to produce more sounds

(B) animals without vocal cords tend to produce as much sound as those with vocal cords

(C) highly intelligent animals tend to produce more sound than less intelligent species

(D) the absence of vocal cords has hindered the adaptation of cetaceans

(E) sound is an important means of communication among whales

         *Like her white friends Eleanor Roosevelt 
         and Aubrey Williams, Mary Bethune believed 
         in the fundamental commitment of the New 
Line Deal to assist the black American’s struggle 
  (5) and in the need for blacks to assume responsibilities 
         to help win that struggle. Unlike 
         those of her white liberal associates, however, 
         Bethune’s ideas had evolved out of a long 
         experience as a “race leader.” Founder of a 
 (10) small black college in Florida, she had become 
         widely known by 1935 as an organizer of 
         black women’s groups and as a civil and political 
         rights activist. Deeply religious, certain of 
         her own capabilities, she held a relatively 
 (15) uncluttered view of what she felt were the 
         New Deal’s and her own people’s obligations 
         to the cause of racial justice. Unafraid to speak 
         her mind to powerful whites, including the 
         President, or to differing black factions, she 
 (20) combined faith in the ultimate willingness of 
         whites to discard their prejudice and bigotry 
         with a strong sense of racial pride and commitment 
         to Negro self-help.

         More than her liberal white friends, Bethune 
 (25) argued for a strong and direct black voice in initiating 
         and shaping government policy. She 
         pursued this in her conversations with President 
         Roosevelt, in numerous memoranda to Aubrey 
         Williams, and in her administrative work as 
 (30) head of the National Youth Administration’s 
         Office of Negro Affairs. With the assistance of 
         Williams, she was successful in having blacks 
         selected to NYA posts at the national, state, and 
         local levels. But she also wanted a black presence 
 (35) throughout the federal government. At the 
         beginning of the war she joined other black 
         leaders in demanding appointments to the 
         Selective Service Board and to the Department 
         of the Army; and she was instrumental in 1941 
 (40) in securing Earl Dickerson’s membership on the 
         Fair Employment Practices Committee. By 
         1944, she was still making appeals for black representation 
         in “all public programs, federal, 
         state, and local,” and “in policy-making posts as 
 (45) well as rank and file jobs.”

         Though recognizing the weakness in the 
         Roosevelt administration’s response to Negro 
         needs, Mary Bethune remained in essence a 
         black partisan champion of the New Deal during 
 (50) the 1930s and 1940s. Her strong advocacy 
         of administration policies and programs was 
         predicated on a number of factors: her assessment 
         of the low status of black Americans during 
         the Depression; her faith in the willingness 
 (55) of some liberal whites to work for the inclusion 
         of blacks in the government’s reform and recovery 
         measures; her conviction that only massive 
         federal aid could elevate the Negro economically; 
         and her belief that the thirties and forties 
 (60) were producing a more self-aware and selfassured 
         black population. Like a number of her 
         white friends in government, Bethune assumed 
         that the preservation of democracy and black 
         people’s “full integration into the benefits and 
 (65) the responsibilities” of American life were inextricably 
         tied together. She was convinced that, 
         with the help of a friendly government, a militant, 
         aggressive “New Negro” would emerge 
         out of the devastation of depression and war, a 
 (70) “New Negro” who would “save America from 
         itself,” who would lead America toward the full 
         realization of its democratic ideas.

14. The author’s main purpose in this passage is to

(A) criticize Mary Bethune for adhering too closely to New Deal policies

(B) argue that Mary Bethune was too optimistic in her assessment of race relations

(C) demonstrate Mary Bethune’s influence on black progress during the Roosevelt years

(D) point out the weaknesses of the white liberal approach to black needs

(E) summarize the attainments of blacks under the auspices of Roosevelt’s New Deal

15. It can be inferred from the passage that Aubrey Williams was which of the following?

  I. A man with influence in the National Youth Administration

 II. A white liberal

III. A man of strong religious convictions

(A) I only

(B) II only

(C) I and II only

(D) II and III only

(E) I, II, and III

16. The author mentions Earl Dickerson (line 40) primarily in order to

(A) cite an instance of Bethune’s political impact

(B) contrast his career with that of Bethune

(C) introduce the subject of a subsequent paragraph

(D) provide an example of Bethune’s “New Negro”

(E) show that Dickerson was a leader of his fellow blacks

17. It can be inferred from the passage that Bethune believed the “New Negro” would “save America from itself” (lines 70–71) by

(A) joining the army and helping America overthrow its Fascist enemies

(B) helping America accomplish its egalitarian ideals

(C) voting for administration antipoverty programs

(D) electing other blacks to government office

(E) expressing a belief in racial pride

18. The tone of the author’s discussion of Bethune is best described as

(A) deprecatory

(B) sentimental

(C) ironic

(D) objective

(E) recriminatory

19. The author uses all the following techniques in the passage EXCEPT

(A) comparison and contrast

(B) development of an extended analogy

(C) direct quotation

(D) general statement and concrete examples

(E) reiteration of central ideas

20. Which of the following statements about the New Deal does the passage best support?

(A) It was strongly committed to justice for all races.

(B) It encouraged black participation in making policy decisions.

(C) It was actively involved in military strategy.

(D) It was primarily the province of Eleanor Roosevelt.

(E) It shaped programs for economic aid and growth.

Reading Comprehension Exercise D

         “The emancipation of women,” James Joyce 
         told one of his friends, “has caused the greatest 
         revolution in our time in the most important 
Line relationship there is — that between men and 
  (5) women.” Other modernists agreed: Virginia 
         Woolf, claiming that in about 1910 “human 
         character changed,” and, illustrating the new 
         balance between the sexes, urged, “Read 
         the ‘Agamemnon,’ and see whether…your, 
 (10) sympathies are not almost entirely with 
         Clytemnestra.” D.H. Lawrence wrote, “perhaps 
         the deepest fight for 2000 years and more, 
         has been the fight for women’s independence.”

         But if modernist writers considered 
 (15) women’s revolt against men’s domination one 
         of their “greatest” and “deepest” themes, only 
         recently — in perhaps the past 15 years — has 
         literary criticism begun to catch up with it. 
         Not that the images of sexual antagonism that 
 (20) abound in modern literature have gone unremarked; 
         far from it. But what we are able to 
         see in literary works depends on the perspectives 
         we bring to them, and now that 
         women — enough to make a difference — 
 (25) are reforming canons and interpreting literature, 
         the landscapes of literary history and the 
         features of individual books have begun to 
         change.

1. According to the passage, women are changing literary criticism by

(A) noting instances of hostility between men and women

(B) seeing literature from fresh points of view

(C) studying the works of early twentieth-century writers

(D) reviewing books written by feminists

(E) resisting masculine influence

2. The author quotes James Joyce, Virginia Woolf, and D.H. Lawrence primarily in order to show that

(A) these were feminist writers

(B) although well-meaning, they were ineffectual

(C) before the twentieth century, there was little interest in women’s literature

(D) modern literature is dependent on the women’s movement

(E) the interest in feminist issues is not new

3. The author’s attitude toward women’s reformation of literary canons can best be described as one of

(A) ambivalence

(B) antagonism

(C) indifference

(D) endorsement

(E) skepticism

4. Which of the following titles best describes the content of the passage?

(A) Modernist Writers and the Search for Equality

(B) The Meaning of Literary Works

(C) Toward a New Criticism

(D) Women in Literature, from 1910 On

(E) Transforming Literature

         Ocean water plays an indispensable role in 
         supporting life. The great ocean basins hold 
         about 300 million cubic miles of water. From 
Line this vast amount, about 80,000 cubic miles of 
  (5) water are sucked into the atmosphere each 
         year by evaporation and returned by precipitation 
         and drainage to the ocean. More than 
         24,000 cubic miles of rain descend annually 
         upon the continents. This vast amount is 
 (10) required to replenish the lakes and streams, 
         springs and water tables on which all flora and 
         fauna are dependent. Thus, the hydrosphere 
         permits organic existence.

         The hydrosphere has strange characteristics 
 (15) because water has properties unlike those of 
         any other liquid. One anomaly is that water 
         upon freezing expands by about 9 percent, 
         whereas most liquids contract on cooling. For 
         this reason, ice floats on water bodies instead 
 (20) of sinking to the bottom. If the ice sank, the 
         hydrosphere would soon be frozen solidly, 
         except for a thin layer of surface melt water 
         during the summer season. Thus, all aquatic 
         life would be destroyed and the interchange of 
 (25) warm and cold currents, which moderates climate, 
         would be notably absent.

         Another outstanding characteristic of water 
         is that water has a heat capacity which is the 
         highest of all liquids and solids except ammonia. 
 (30) This characteristic enables the oceans to 
         absorb and store vast quantities of heat, 
         thereby often preventing climatic extremes. In 
         addition, water dissolves more substances 
         than any other liquid. It is this characteristic 
 (35) which helps make oceans a great storehouse 
         for minerals which have been washed down 
         from the continents. In several areas of the 
         world these minerals are being commercially 
         exploited. Solar evaporation of salt is widely 
 (40) practiced, potash is extracted from the Dead 
         Sea, and magnesium is produced from sea 
         water along the American Gulf Coast.

5. The author’s main purpose in this passage is to

(A) describe the properties and uses of water

(B) illustrate the importance of conserving water

(C) explain how water is used in commerce and industry

(D) reveal the extent of the earth’s ocean masses

(E) compare water with other liquids

6. According to the passage, fish can survive in the oceans because

(A) they do not need oxygen

(B) ice floats

(C) evaporation and condensation create a water cycle

(D) there are currents in the oceans

(E) water absorbs heat

7. Which of the following characteristics of water does the author mention in the passage?

  I. Water expands when it is frozen.

 II. Water is a good solvent.

III. Water can absorb heat.

(A) I only

(B) II only

(C) I and II only

(D) II and III only

(E) I, II, and III

8. According to the passage, the hydrosphere is NOT

(A) responsible for all forms of life

(B) able to modify weather

(C) a source of natural resources

(D) in danger of freezing over

(E) the part of the earth covered by water

9. The author’s tone in the passage can best be described as

(A) dogmatic

(B) dispassionate

(C) speculative

(D) biased

(E) hortatory

10. The author organizes the passage by

(A) comparison and contrast

(B) juxtaposition of true and untrue ideas

(C) general statements followed by examples

(D) hypothesis and proof

(E) definition of key terms

11. Which of the following statements would be most likely to begin the paragraph immediately following the passage?

(A) Water has the ability to erode the land.

(B) Magnesium is widely used in metallurgical processes.

(C) Now let us consider the great land masses.

(D) Another remarkable property of ice is its strength.

(E) Droughts and flooding are two types of disasters associated with water.

         The opposite of adaptive divergence is an 
         interesting and fairly common expression of 
         evolution. Whereas related groups of organisms 
Line take on widely different characters in 
  (5) becoming adapted to unlike environments in 
         the case of adaptive divergence, we find that 
         unrelated groups of organisms exhibit adaptive 
         convergence when they adopt similar 
         modes of life or become suited for special 
 (10) sorts of environments. For example, invertebrate 
         marine animals living firmly attached to 
         the sea bottom or to some foreign object tend 
         to develop a subcylindrical or conical form. 
         This is illustrated by coral individuals, by 
 (15) many sponges, and even by the diminutive 
         tubes of bryozoans. Adaptive convergence in 
         taking this coral-like form is shown by some 
         brachiopods and pelecypods that grew in 
         fixed position. More readily appreciated is the 
 (20) streamlined fitness of most fishes for moving 
         swiftly through water; they have no neck, the 
         contour of the body is smoothly curved so as 
         to give minimum resistance, and the chief 
         propelling organ is a powerful tail fin. The 
 (25) fact that some fossil reptiles (ichthyosaurs) 
         and modern mammals (whales, dolphins) are 
         wholly fishlike in form is an expression of 
         adaptive convergence, for these air-breathing 
         reptiles and mammals, which are highly efficient 
 (30) swimmers, are not closely related to 
         fishes. Unrelated or distantly related organisms 
         that develop similarity of form are sometimes 
         designated as homeomorphs (having 
         the same form).

12. The author mentions ichthyosaurs and dolphins (lines 25 and 26) as examples of

(A) modern mammalian life forms that are aquatic

(B) species of slightly greater mobility than brachiopods

(C) air-breathing reptiles closely related to fish

(D) organisms that have evolved into fishlike forms

(E) invertebrate and vertebrate marine animals

13. According to the passage, adaptive convergence and adaptive divergence are

(A) manifestations of evolutionary patterns

(B) hypotheses unsupported by biological phenomena

(C) ways in which plants and animals adjust to a common environment

(D) demonstrated by brachiopods and pelecypods

(E) compensatory adjustments made in response to unlike environments

14. It can be inferred that in the paragraph immediately preceding this passage the author discussed

(A) marine intelligence

(B) adaptive divergence

(C) air-breathing reptiles

(D) environmental impacts

(E) organisms with similar forms

         Nearly two thousand years have passed 
         since a census decreed by Caesar Augustus 
         became part of the greatest story every told. 
Line Many things have changed in the intervening 
  (5) years. The hotel industry worries more about 
         overbuilding than overcrowding, and if they 
         had to meet an unexpected influx, few inns 
         would have a manger to accommodate the 
         weary guests. Now it is the census taker that 
 (10) does the traveling in the fond hope that a 
         highly mobile population will stay put long 
         enough to get a good sampling. Methods of 
         gathering, recording, and evaluating information 
         have presumably been improved a great 
 (15) deal. And where then it was the modest purpose 
         of Rome to obtain a simple head count as 
         an adequate basis for levying taxes, now batteries 
         of complicated statistical series furnished 
         by governmental agencies and private organizations 
 (20) are eagerly scanned and interpreted by 
         sages and seers to get a clue to future events. 
         The Bible does not tell us how the Roman 
         census takers made out, and as regards our 
         more immediate concern, the reliability of 
 (25) present-day economic forecasting, there are 
         considerable differences of opinion. They were 
         aired at the celebration of the 125th anniversary 
         of the American Statistical Association. 
         There was the thought that business forecasting 
 (30) might well be on its way from an art to a 
         science, and some speakers talked about newfangled 
         computers and high-falutin mathematical 
         systems in terms of excitement and 
         endearment which we, at least in our younger 
 (35) years when these things matter, would have 
         associated more readily with the description of 
         a fair maiden. But others pointed to the 
         deplorable record of highly esteemed forecasts 
         and forecasters with a batting average below 
 (40) that of the Mets, and the president-elect of the 
         Association cautioned that "high powered statistical 
         methods are usually in order where the 
         facts are crude and inadequate, the exact contrary 
         of what crude and inadequate statisticians 
 (45) assume.” We left this birthday party 
         somewhere between hope and despair and 
         with the conviction, not really newly acquired, 
         that proper statistical methods applied to 
         ascertainable facts have their merits in economic 
 (50) forecasting as long as neither forecaster 
         nor public is deluded into mistaking the delineation 
         of probabilities and trends for a prediction 
         of certainties of mathematical exactitude.

15. The passage would be most likely to appear in

(A) a journal of biblical studies

(B) an introductory college textbook on statistics

(C) the annual report of the American Statistical Association

(D) a newspaper review of a recent professional festivity

(E) the current bulletin of the census bureau

16. According to the passage, taxation in Roman times was based on

(A) mobility

(B) wealth

(C) population

(D) census takers

(E) economic predictions

17. The author refers to the Romans primarily in order to

(A) prove the superiority of modern sampling methods to ancient ones provide a historical framework for the passage

(C) relate an unfamiliar concept to a familiar one

(D) show that statistical forecasts have not significantly deteriorated

(E) cite an authority to support the thesis of the passage

18. The author refers to the Mets primarily in order to

(A) show that sports do not depend on statistics

(B) provide an example of an unreliable statistic

(C) contrast verifiable and unverifiable methods of record keeping

(D) indicate the changes in attitudes from Roman days to the present

(E) illustrate the failure of statistical predictions

19. On the basis of the passage, it can be inferred that the author would agree with which of the following statements?

(A) Computers have significantly improved the application of statistics in business.

(B) Statistics is not, at the present time, a science.

(C) It is useless to try to predict the economy.

(D) Most mathematical systems are inexact.

(E) Statisticians should devote themselves to the study of probability.

20. The author’s tone can best be described as

(A) jocular

(B) scornful

(C) pessimistic

(D) objective

(E) humanistic

Reading Comprehension Exercise E

         Observe the dilemma of the fungus: it is a 
         plant, but it possesses no chlorophyll. While 
         all other plants put the sun's energy to work for 
Line them combining the nutrients of ground and 
  (5) air into the body structure, the chlorophylless 
         fungus must look elsewhere for an energy supply. 
         It finds it in those other plants which, having 
         received their energy free from the sun, 
         relinquish it at some point in their cycle either 
 (10) to animals (like us humans) or to fungi.

         In this search for energy the fungus has 
         become the earth's major source of rot and 
         decay. Wherever you see mold forming on a 
         piece of bread, or a pile of leaves turning to 
 (15) compost, or a blown-down tree becoming pulp 
         on the ground, you are watching a fungus eating. 
         Without fungus action the earth would be 
         piled high with the dead plant life of past centuries. 
         In fact, certain plants which contain 
 (20) resins that are toxic to fungi will last indefinitely; 
         specimens of the redwood, for instance, 
         can still be found resting on the forest floor 
         centuries after having been blown down.

1. Which of the following words best describes the fungus as depicted in the passage?

(A) Unevolved

(B) Sporadic

(C) Enigmatic

(D) Parasitic

(E) Toxic

2. The passage states all the following about fungi EXCEPT:

(A) They are responsible for the decomposition of much plant life.

(B) They cannot live completely apart from other plants.

(C) They are vastly different from other plants.

(D) They are poisonous to resin-producing plants.

(E) They cannot produce their own store of energy.

3. The author’s statement that “you are watching a fungus eating” (lines 16–17) is best described as

(A) figurative

(B) ironical

(C) parenthetical

(D) erroneous

(E) contradictory

4. The author is primarily concerned with

(A) warning people of the dangers of fungi

(B) writing a humorous essay on fungi

(C) relating how most plants use solar energy

(D) describing the actions of fungi

(E) explaining the long life of some redwoods

         The establishment of the Third Reich 
         influenced events in American history by 
         starting a chain of events which culminated in 
Line war between Germany and the United States. 
  (5) The complete destruction of democracy, the 
         persecution of Jews, the war on religion, the 
         cruelty and barbarism of the Nazis, and especially 
         the plans of Germany and her allies, 
         Italy and Japan, for world conquest caused 
 (10) great indignation in this country and brought 
         on fear of another world war. While speaking 
         out against Hitler's atrocities, the American 
         people generally favored isolationist policies 
         and neutrality. The Neutrality Acts of 1935 
 (15) and 1936 prohibited trade with any belligerents 
         or loans to them. In 1937 the President 
         was empowered to declare an arms embargo 
         in wars between nations at his discretion.

         American opinion began to change somewhat 
 (20) after President Roosevelt's "quarantine 
         the aggressor" speech at Chicago (1937), in 
         which he severely criticized Hitler's policies. 
         Germany's seizure of Austria and the Munich 
         Pact for the partition of Czechoslovakia 
 (25) (1938) also aroused the American people. The 
         conquest of Czechoslovakia in March 1939 
         was another rude awakening to the menace of 
         the Third Reich. In August 1939 came the 
         shock of the Nazi-Soviet Pact and in 
 (30) September the attack on Poland and the outbreak 
         of European war. The United States 
         attempted to maintain neutrality in spite of 
         sympathy for the democracies arrayed against 
         the Third Reich. The Neutrality Act of 1939 
 (35) repealed the arms embargo and permitted 
         "cash and carry" exports of arms to belligerent 
         nations. A strong national defense program 
         was begun. A draft act was passed (1940) to 
         strengthen the military services. A Lend-Lease 
 (40) Act (1941) authorized the President to sell, 
         exchange, or lend materials to any country 
         deemed necessary by him for the defense of 
         the United States. Help was given to Britain 
         by exchanging certain overage destroyers for 
 (45) the right to establish American bases in 
         British territory in the Western Hemisphere. 
         In August 1941 President Roosevelt and 
         Prime Minister Churchill met and issued the 
         Atlantic Charter, which proclaimed the kind 
 (50) of a world that should be established after the 
         war. In December 1941 Japan launched an 
         unprovoked attack on the United States at Pearl 
         Harbor. Immediately thereafter, Germany 
         declared war on the United States.

5. The author is primarily concerned with

(A) evaluating various legislative efforts to strengthen national defense

(B) summarizing the events that led up to America’s involvement in the war

(C) criticizing the atrocities perpetrated by the Third Reich

(D) explaining a basic distinction between American and German policy

(E) describing the social and psychological effects of war

6. During the years 1933–36, American foreign policy may best be described as being one of

(A) overt belligerence

(B) deliberate uninvolvement

(C) moral indignation

(D) veiled contempt

(E) reluctant admiration

7. According to the passage, the United States, while maintaining neutrality, showed its sympathy for the democracies by which of the following actions?

  I. It came to the defense of Poland.

 II. It conscripted recruits for the armed forces.

III. It supplied weapons to friendly countries.

(A) I only

(B) III only

(C) I and II only

(D) II and III only

(E) I, II, and III

8. According to the passage, all of the following events occurred in 1939 EXCEPT

(A) the invasion of Poland

(B) the invasion of Czechoslovakia

(C) the annexation of Austria

(D) passage of the Neutrality Act

(E) the beginning of the war in Europe

9. With which of the following statements would the author of the passage be most likely to agree?

(A) American neutrality during the 1930s was a natural consequence of the course of world events.

(B) Every nation should be free to determine its own internal policy without interference.

(C) The United States, through its aggressive actions, invited an attack on its territory.

(D) Americans were slow to realize the full danger posed by Nazi Germany.

(E) President Roosevelt showed undue sympathy for Britain.

10. Which of the following best describes the organization of the passage?

(A) The author presents a thesis and then lists events that support that thesis in chronological order.

(B) The author presents a thesis and then cites examples that support the thesis as well as evidence that tends to negate it.

(C) The author summarizes a historical study and then discusses an aspect of the study in detail.

(D) The author describes historical events and then gives a personal interpretation of them.

(E) The author cites noted authorities as a means of supporting his or her own opinion.

         Not a few of Jane Austen's personal acquaintances 
         might have echoed Sir Samuel Egerton 
         Brydges, who noticed that “she was fair and 
Line handsome, slight and elegant, but with cheeks 
  (5) a little too full,” while “never suspect[ing] she 
         was an authoress.” For this novelist whose personal 
         obscurity was more complete than that 
         of any other famous writer was always quick to 
         insist either on complete anonymity or on the 
 (10) propriety of her limited craft, her delight in 
         delineating just “3 or 4 Families in a Country 
         Village.” With her self-deprecatory remarks 
         about her inability to join “strong manly, spirited 
         sketches, full of Variety and Glow” with 
 (15) her “little bit (two Inches wide) of Ivory,” Jane 
         Austen perpetuated the belief among her 
         friends that her art was just an accomplishment 
         "by a lady,” if anything “rather too light 
         and bright and sparkling.” In this respect she 
 (20) resembled one of her favorite contemporaries, 
         Mary Brunton, who would rather have 
         "glid[ed] through the world unknown” than 
         been “sus pected of literary airs — to be 
         shunned, as literary women are, by the more 
 (25) pretending of their own sex, and abhorred, as 
         literary women are, by the more pretending of 
         the other! — my dear, I would sooner exhibit 
         as a ropedancer.” 
               Yet, decorous though they might first seem, 
 (30) Austen's self-effacing anonymity and her modest 
         description of her miniaturist art also imply 
         a criticism, even a rejection, of the world at 
         large. For, as Gaston Bachelard explains, the 
         miniature “allows us to be world conscious at 
 (35) slight risk.” While the creators of satirically 
         conceived diminutive landscapes seem to see 
         everything as small because they are themselves 
         so grand, Austen's analogy for her art— 
         her “little bit (two Inches wide) of Ivory”— 
 (40) suggests a fragility that reminds us of the risk 
         and instability outside the fictional space. 
         Besides seeing her art metaphorically, as her 
         critics would too, in relation to female arts 
         severely devalued until quite recently (for 
 (45) painting on ivory was traditionally a “ladylike” 
         occupation), Austen attempted through selfimposed 
         novelistic limitations to define a 
         secure place, even as she seemed to admit the 
         impossibility of actually inhabiting such a 
 (50) small space with any degree of comfort. And 
         always, for Austen, it is women- because 
         they are too vulnerable in the world at large- 
         who must acquiesce in their own confinement, 
         no matter how stifling it may be.

11. The passage focuses primarily on

(A) Jane Austen’s place in English literature

(B) the literary denigration of female novelists

(C) the implications of Austen’s attitude to her work

(D) critical evaluations of the novels of Jane Austen

(E) social rejection of professional women in the 18th and 19th centuries

12. According to the passage, Austen concentrated on a limited range of subjects because

(A) she had a limited degree of experience of fiction

(B) her imagination was incapable of creating other worlds

(C) women in her time were prohibited from writing about significant topics

(D) she wanted to create a safe niche for the exercise of her talents

(E) she did not wish to be acknowledged as an author

13. Which of the following best expresses the relationship of the first sentence to the rest of the passage?

(A) Specific instance followed by generalizations

(B) Assertion followed by analysis

(C) Objective statement followed by personal opinion

(D) Quotation from an authority followed by conflicting views

(E) Challenge followed by debate

         The atmosphere is a mixture of several 
         gases. There are about ten chemical elements 
         which remain permanently in gaseous form in 
Line the atmosphere under all natural conditions. 
  (5) Of these permanent gases, oxygen makes up 
         about 21 percent and nitrogen about 78 percent. 
         Several other gases, such as argon, carbon 
         dioxide, hydrogen, neon, krypton, and 
         xenon, comprise the remaining 1 percent of 
 (10) the volume of dry air. The amount of water 
         vapor, and its variations in amount and distribution, 
         are of extraordinary importance in 
         weather changes. Atmospheric gases hold in 
         suspension great quantities of dust, pollen, 
 (15) smoke, and other impurities which are always 
         present in considerable, but variable amounts. 
               The atmosphere has no definite upper limits 
         but gradually thins until it becomes imperceptible. 
         Until recently it was assumed that 
 (20) the air above the first few miles gradually grew 
         thinner and colder at a constant rate. It was 
         also assumed that upper air had little influence 
         on weather changes. Recent studies of 
         the upper atmosphere, currently being conducted 
 (25) by earth satellites and missile probings, 
         have shown these assumptions to be 
         incorrect. The atmosphere has three welldefined 
         strata. 
               The layer of the air next to the earth, which 
 (30) extends upward for about 10 miles, is known 
         as the troposphere. On the whole, it makes up 
         about 75 percent of all the weight of the 
         atmosphere. It is the warmest part of the 
         atmosphere because most of the solar radiation 
 (35) is absorbed by the earth’s surface, which warms 
         the air immediately surrounding it. A steady 
         decrease of temperature with increasing elevation 
         is a most striking characteristic. The 
         upper layers are colder because of their greater 
 (40) distance from the earth’s surface and rapid 
         radiation of heat into space. The temperatures 
         within the troposphere decrease about 3.5 
         degrees per 1000-foot increase in altitude. 
         Within the troposphere, winds and air currents 
 (45) distribute heat and moisture. Strong 
         winds, called jet streams, are located at the 
         upper levels of the troposphere. These jet 
         streams are both complex and widespread in 
         occurrence. They normally show a waveshaped 
 (50) pattern and move from west to east at velocities 
         of 150 mph, but velocities as high as 400 
         mph have been noted. The influences of 
         changing locations and strengths of jet streams 
         upon weather conditions and patterns are no 
 (55) doubt considerable. Current intensive research 
         may eventually reveal their true significance. 
               Above the troposphere to a height of about 
         50 miles is a zone called the stratosphere. The 
         stratosphere is separated from the troposphere 
 (60) by a zone of uniform temperatures called the 
         tropopause. Within the lower portions of the 
         stratosphere is a layer of ozone gases which filters 
         out most of the ultraviolet rays from the 
         sun. The ozone layer varies with air pressure. 
 (65) If this zone were not there, the full blast of the 
         sun's ultraviolet light would burn our skins, 
         blind our eyes, and eventually result in our 
         destruction. Within the stratosphere, the temperature 
         and atmospheric composition are relatively 
 (70) uniform. 
               The layer upward of about 50 miles is the 
         most fascinating but the least known of these 
         three strata. It is called the ionosphere because 
         it consists of electrically charged particles 
 (75) called ions, thrown from the sun. The northern 
         lights (aurora borealis) originate within 
         this highly charged portion of the atmosphere. 
         Its effect upon weather conditions, if 
         any, is as yet unknown.

14. Which of the following titles best summarizes the content of the passage?

(A) New Methods for Calculating the Composition of the Atmosphere

(B) New Evidence Concerning the Stratification of the Atmosphere

(C) The Atmosphere: Its Nature and Importance to Our Weather

(D) The Underlying Causes of Atmospheric Turbulence

(E) Stratosphere, Troposphere, Ionosphere: Three Similar Zones

15. The passage supplies information that would answer which of the following questions?

  I. How do the troposphere and the stratosphere differ?

 II. How does the ionosphere affect the weather?

III. How do earth satellites study the atmosphere?

(A) I only

(B) III only

(C) I and II only

(D) I and III only

(E) I, II, and III

16. According to the passage, life as we know it exists on earth because the atmosphere

(A) contains a layer of ozone gases

(B) contains electrically charged particles

(C) is warmest at the bottom

(D) carries the ultraviolet rays of the sun

(E) provides the changes in weather

17. It can be inferred from the passage that a jet plane will usually have its best average rate of speed on its run from

(A) New York to San Francisco

(B) Los Angeles to New York

(C) Boston to Miami

(D) Bermuda to New York

(E) London to Washington, D.C.

18. It can be inferred from the passage that at the top of Jungfrau, which is 12,000 feet above the town of Interlaken in Switzerland, the temperature is usually

(A) below freezing

(B) about 42 degrees colder than on the ground

(C) warmer than in Interlaken

(D) affected by the ionosphere

(E) about 75 degrees colder than in Interlaken

19. The passage states that the troposphere is the warmest part of the atmosphere because it

(A) is closest to the sun

(B) contains electrically charged particles

(C) radiates heat into space

(D) has winds and air current that distribute the heat

(E) is warmed by the earth’s heat

20. According to the passage, the atmosphere consists of all of the following EXCEPT

(A) 21 percent oxygen

(B) a definite amount of water vapor

(C) ten permanent elements

(D) less than 1 percent xenon

(E) considerable waste products

Answer Key

Reading Comprehension Exercise A

1. B   

6. C   

11. C   

16. D

2. B   

7. E   

12. D   

17. D

3. E   

8. A   

13. D   

18. C

4. B   

9. E   

14. A   

19. E

5. E   

10. D   

15. B   

20. E

Reading Comprehension Exercise B

1. A

6. E

11. D

16. B

2. D

7. C

12. C

17. A

3. C

8. E

13. E

18. C

4. B

9. C

14. B

19. E

5. D

10. C

15. A

20. B

Reading Comprehension Exercise C

1. C   

6. B   

11. C   

16. A

2. B   

7. A   

12. D   

17. B

3. A   

8. B   

13. A   

18. D

4. D   

9. D   

14. C   

19. B

5. E   

10. B   

15. C   

20. E

Reading Comprehension Exercise D

1. B   

6. B   

11. A   

16. C

2. E   

7. E   

12. D   

17. B

3. D   

8. D   

13. A   

18. E

4. C   

9. B   

14. B   

19. B

5. A   

10. C   

15. D   

20. A

Reading Comprehension Exercise E

1. D   

6. B   

11. C   

16. A

2. D   

7. D   

12. D   

17. B

3. A   

8. C   

13. B   

18. B

4. D   

9. D   

14. C   

19. E

5. B   

10. A   

15. A   

20. B

1Edwin O. Wilson, Harvard professor and author of Sociobiology.

*Note that this passage is representative of the time it discusses, and therefore uses the terminology commonly accepted in that period.